Q11

 
SandiG694
Thanks Received: 0
Vinny Gambini
Vinny Gambini
 
Posts: 2
Joined: September 07th, 2020
 
 
 

Q11

by SandiG694 Sat Dec 19, 2020 9:09 pm

Can someone please explain Q11
 
andreperez7
Thanks Received: 3
Jackie Chiles
Jackie Chiles
 
Posts: 45
Joined: March 11th, 2013
 
 
 

Re: Q11

by andreperez7 Fri Dec 25, 2020 5:13 pm

I think the order of operation here is the following:

1) Use previous work to see if the AC held under the old rule. If it does it's a contender; if not, it's out.
We see that answers C-E all can be broken under the old rule, so they're not valid substitutes.

2) Inspect A and B. Either show that they allow for scenarios not possible under the original rule, or restrict scenarios possible under the old rule to strike them out.

3) Answer A says, if H/1, then G/3.
While if H is in 1, then certainly J and G are separate because J can't into 3. However, if H is 3, then both G and J can go into 2. That's not allowed under the old rule. So A is out.

4) At this point you should pick B and move on, but for posterity let's figure out answer choice B.
If J is in, then G is in 3 and that separates G and J. But what if G is in 2, the only other position it can go in?
If G/2 then J can't go into 2, because of the contrapositive of the rule (~G/3 -> ~J/2), which means J/1. In effect, G and J can't go together. This is the correct answer choice.

I hope this was of help.
 
Misti Duvall
Thanks Received: 13
Atticus Finch
Atticus Finch
 
Posts: 191
Joined: June 23rd, 2016
 
 
 

Re: Q11

by Misti Duvall Thu Jan 07, 2021 12:59 am

Not bad, Andre!

Rule substitution questions are the worst, and the answer will rely on the function the original rule plays in the game. Here's how I did it: since we know that G can't be in 1 and J can't be in 3, the only possible place they could be together is group 2. So if there's a rule that would force them apart in group 2, that's pretty good. Answer choice (B) does that.
LSAT Instructor | Manhattan Prep